Fourier Transform of Heaviside function

Click For Summary
The discussion centers on finding the Fourier transform of the function H(x-a)e^{-bx}, where H(x) is the Heaviside function. The initial attempts at solving the problem involved integrating from a to infinity, but the user faced confusion regarding variable consistency and the correct application of Fourier transform equations. Clarification was provided that the variable should consistently be x, not t, to properly set up the Fourier transform. The user expressed uncertainty about the integration process and whether their approach would yield the correct result. The conversation highlights the importance of maintaining variable clarity in mathematical transformations.
EdisT
Messages
17
Reaction score
0

Homework Statement


Find the Fourier transform of H(x-a)e^{-bx}, where H(x) is the Heaviside function.

Homework Equations


\mathcal{F}[f(t)]=\frac{1}{2 \pi} \int_{- \infty}^{\infty} f(t) \cdot e^{-i \omega t} dt
Convolution theory equations that might be relevant:
\mathcal{F}[f(t) \cdot g(t)] = \mathcal{F}[f(t)] * \mathcal{F}[g(t)]
\mathcal{F}[e^{\alpha t} \cdot f(t)] = \mathcal{F}[f(t+\alpha t)]
Derivative of the Heaviside function:
H'(t) = \delta(t)
where \delta(t) is the Dirac Delta function.

The Attempt at a Solution


Using the first relevant equation, and assuming the Heaviside function simply changed the boundary conditions from a to infinity:
\mathcal{F}[H(x-a)e^{-bx}]=\frac{1}{2 \pi} \int_{a}^{\infty} e^{-bx} \cdot e^{-i \omega t} dt
= - \frac{1}{2 \pi} \left[\frac{i e^{-bx-i \omega t}}{\omega} \right]_{a}^{\infty} = - \frac{i e^{-bx-i \omega a}}{\sqrt{2\pi} \omega}

This was not right so I tried to take the integral of the Heaviside function via integration by parts:
\mathcal{F}[f(t)]=\frac{1}{2 \pi} \int_{- \infty}^{\infty}H(x-a)e^{-bx-i \omega t} dt
=\frac{1}{2 \pi} \left[\frac{-H(x-a)ie^{-bx-i \omega t}}{\omega} \right]_{-\infty}^{\infty} - \int_{\infty}^{\infty} \delta(x-a)e^{-bx-i \omega t}
At this point I could go on with the integration however I'm unsure if it'll lead to the right answer or if what I'm doing makes sense. I have considered using the last equation in part of the convolution theory section, to get an integral of perhaps H(x-a-i \omega t) but that doesn't seem right either.. I am unsure how to take the Fourier transformation of the Heaviside function, any help will be gladly appreciated.
 
Physics news on Phys.org
You are mixing variables. Either use x or t, for this problem it is quite obvious that x is the variable and you need to set up your Fourier transform accordingly. (If it was in t, then your function would be a constant and the Heaviside would not change the integration domain.)
 
Would it then just be the integral from a to infinity of H(x-a)e^{-bx-i \omega x} dx or the integral from a to infinity of e^{-bx-i \omega x} ?
 
EdisT said:
Would it then just be the integral from a to infinity of H(x-a)e^{-bx-i \omega x} dx or the integral from a to infinity of e^{-bx-i \omega x} ?

Aren't these exactly the same thing?
 
Question: A clock's minute hand has length 4 and its hour hand has length 3. What is the distance between the tips at the moment when it is increasing most rapidly?(Putnam Exam Question) Answer: Making assumption that both the hands moves at constant angular velocities, the answer is ## \sqrt{7} .## But don't you think this assumption is somewhat doubtful and wrong?

Similar threads

Replies
1
Views
2K
  • · Replies 3 ·
Replies
3
Views
3K
  • · Replies 6 ·
Replies
6
Views
3K
  • · Replies 3 ·
Replies
3
Views
2K
  • · Replies 2 ·
Replies
2
Views
3K
  • · Replies 1 ·
Replies
1
Views
2K
  • · Replies 1 ·
Replies
1
Views
1K
Replies
2
Views
2K
  • · Replies 31 ·
2
Replies
31
Views
4K
  • · Replies 3 ·
Replies
3
Views
1K